Prove that $K$ and $L$ are conjugate iff $G(E/K)$ and $G(E/L)$ are conjugate subgroups of $G(E/F)$

The name of the pictureThe name of the pictureThe name of the pictureClash Royale CLAN TAG#URR8PPP











up vote
0
down vote

favorite












Let $E$ be an extension field of $F$ and suppose that $K$ and $L$ are two intermediate fields. If there exists an element $sigma in G(E/F)$ such that $sigma(K) = L$ then K and L are said to be conjugate fields. Prove that $K$ and $L$ are conjugate iff $G(E/K)$ and $G(E/L)$ are conjugate subgroups of $G(E/F)$.



This direction ($Rightarrow$):



Consider $sigma G(E/K) sigma^-1$ where $sigma in G(E/F)$ and $sigma(K) = L$ and consider some element $alpha in L$. I believe that if I can show that $sigma G(E/K) sigma^-1$ fixes $alpha$, then we are done.



So consider any $gamma in G(E/K)$. As $sigma (K) = L$ then $K = sigma^-1(L)$.



Thus, $sigma^-1(alpha)=beta$ for some $betain K$.



And then, $gamma(sigma^-1(alpha))=gamma(beta)=beta$ as $gamma$ fixes all $beta in K$



Finally, $sigma(gamma(sigma^-1(alpha)))=sigma(beta)=alpha$. So, $sigma G(E/K) sigma^-1$ consists of all automorphisms of E that fix $alpha in L$ and so it is $G(E/L)$



This direction ($Leftarrow$):



Well, if $G(E/K)$ and $G(E/L)$ are conjugate then there exists some $sigma in G(E/F)$ such that $sigma G(E/K) sigma^-1 = G(E/L)$. So consider some $alpha in K$. If $alpha$ is also in $F$ (as $K$ is an extension field of $F$ because it is an intermediate field between $E$ and $F$) then $alpha$ is fixed by $sigma G(E/K) sigma^-1$ and as $alpha in F$ then $alpha$ is also in $L$.



If $alpha in Kbackslash F$ then I don't know how to proceed.







share|cite|improve this question























    up vote
    0
    down vote

    favorite












    Let $E$ be an extension field of $F$ and suppose that $K$ and $L$ are two intermediate fields. If there exists an element $sigma in G(E/F)$ such that $sigma(K) = L$ then K and L are said to be conjugate fields. Prove that $K$ and $L$ are conjugate iff $G(E/K)$ and $G(E/L)$ are conjugate subgroups of $G(E/F)$.



    This direction ($Rightarrow$):



    Consider $sigma G(E/K) sigma^-1$ where $sigma in G(E/F)$ and $sigma(K) = L$ and consider some element $alpha in L$. I believe that if I can show that $sigma G(E/K) sigma^-1$ fixes $alpha$, then we are done.



    So consider any $gamma in G(E/K)$. As $sigma (K) = L$ then $K = sigma^-1(L)$.



    Thus, $sigma^-1(alpha)=beta$ for some $betain K$.



    And then, $gamma(sigma^-1(alpha))=gamma(beta)=beta$ as $gamma$ fixes all $beta in K$



    Finally, $sigma(gamma(sigma^-1(alpha)))=sigma(beta)=alpha$. So, $sigma G(E/K) sigma^-1$ consists of all automorphisms of E that fix $alpha in L$ and so it is $G(E/L)$



    This direction ($Leftarrow$):



    Well, if $G(E/K)$ and $G(E/L)$ are conjugate then there exists some $sigma in G(E/F)$ such that $sigma G(E/K) sigma^-1 = G(E/L)$. So consider some $alpha in K$. If $alpha$ is also in $F$ (as $K$ is an extension field of $F$ because it is an intermediate field between $E$ and $F$) then $alpha$ is fixed by $sigma G(E/K) sigma^-1$ and as $alpha in F$ then $alpha$ is also in $L$.



    If $alpha in Kbackslash F$ then I don't know how to proceed.







    share|cite|improve this question





















      up vote
      0
      down vote

      favorite









      up vote
      0
      down vote

      favorite











      Let $E$ be an extension field of $F$ and suppose that $K$ and $L$ are two intermediate fields. If there exists an element $sigma in G(E/F)$ such that $sigma(K) = L$ then K and L are said to be conjugate fields. Prove that $K$ and $L$ are conjugate iff $G(E/K)$ and $G(E/L)$ are conjugate subgroups of $G(E/F)$.



      This direction ($Rightarrow$):



      Consider $sigma G(E/K) sigma^-1$ where $sigma in G(E/F)$ and $sigma(K) = L$ and consider some element $alpha in L$. I believe that if I can show that $sigma G(E/K) sigma^-1$ fixes $alpha$, then we are done.



      So consider any $gamma in G(E/K)$. As $sigma (K) = L$ then $K = sigma^-1(L)$.



      Thus, $sigma^-1(alpha)=beta$ for some $betain K$.



      And then, $gamma(sigma^-1(alpha))=gamma(beta)=beta$ as $gamma$ fixes all $beta in K$



      Finally, $sigma(gamma(sigma^-1(alpha)))=sigma(beta)=alpha$. So, $sigma G(E/K) sigma^-1$ consists of all automorphisms of E that fix $alpha in L$ and so it is $G(E/L)$



      This direction ($Leftarrow$):



      Well, if $G(E/K)$ and $G(E/L)$ are conjugate then there exists some $sigma in G(E/F)$ such that $sigma G(E/K) sigma^-1 = G(E/L)$. So consider some $alpha in K$. If $alpha$ is also in $F$ (as $K$ is an extension field of $F$ because it is an intermediate field between $E$ and $F$) then $alpha$ is fixed by $sigma G(E/K) sigma^-1$ and as $alpha in F$ then $alpha$ is also in $L$.



      If $alpha in Kbackslash F$ then I don't know how to proceed.







      share|cite|improve this question











      Let $E$ be an extension field of $F$ and suppose that $K$ and $L$ are two intermediate fields. If there exists an element $sigma in G(E/F)$ such that $sigma(K) = L$ then K and L are said to be conjugate fields. Prove that $K$ and $L$ are conjugate iff $G(E/K)$ and $G(E/L)$ are conjugate subgroups of $G(E/F)$.



      This direction ($Rightarrow$):



      Consider $sigma G(E/K) sigma^-1$ where $sigma in G(E/F)$ and $sigma(K) = L$ and consider some element $alpha in L$. I believe that if I can show that $sigma G(E/K) sigma^-1$ fixes $alpha$, then we are done.



      So consider any $gamma in G(E/K)$. As $sigma (K) = L$ then $K = sigma^-1(L)$.



      Thus, $sigma^-1(alpha)=beta$ for some $betain K$.



      And then, $gamma(sigma^-1(alpha))=gamma(beta)=beta$ as $gamma$ fixes all $beta in K$



      Finally, $sigma(gamma(sigma^-1(alpha)))=sigma(beta)=alpha$. So, $sigma G(E/K) sigma^-1$ consists of all automorphisms of E that fix $alpha in L$ and so it is $G(E/L)$



      This direction ($Leftarrow$):



      Well, if $G(E/K)$ and $G(E/L)$ are conjugate then there exists some $sigma in G(E/F)$ such that $sigma G(E/K) sigma^-1 = G(E/L)$. So consider some $alpha in K$. If $alpha$ is also in $F$ (as $K$ is an extension field of $F$ because it is an intermediate field between $E$ and $F$) then $alpha$ is fixed by $sigma G(E/K) sigma^-1$ and as $alpha in F$ then $alpha$ is also in $L$.



      If $alpha in Kbackslash F$ then I don't know how to proceed.









      share|cite|improve this question










      share|cite|improve this question




      share|cite|improve this question









      asked Jul 18 at 16:48









      math4everyone

      38918




      38918




















          1 Answer
          1






          active

          oldest

          votes

















          up vote
          0
          down vote













          I believe that the you need the assumption that $F subseteq E$ is a Galois extension. Indeed consider the non-Galois extension $mathbbQ subseteq E = mathbbQ(sqrt[3]2)$. The Galois group is the trivial group of a single element. Hence $G(E/mathbbQ) = G(E/E)$ and in particular they are conjugates. However it is not true that there exists $sigma in G(E/mathbbQ)$ s.t. $sigma(E) = mathbbQ$, as then $sigma$ must be the identity and we know $E not = mathbbQ$



          If we assume the extension is Galois the claim holds. In particular if $sigma G(E/K)sigma^-1 = G(E/L)$, then we have that $sigma(K) = L$.



          First we'll prove $sigma(K) subseteq L$. Consider $sigma(alpha)$ for some $alpha in K$. Also let $gamma$ be an arbitrary element of $G(E/L)$. Then we have that $exists beta in G(E/K)$ s.t. $gamma = sigma beta sigma^-1$. Then we have:



          $$gamma(sigma(alpha)) = sigma beta sigma^-1(sigma(alpha)) = sigma(alpha)$$



          So we have that $sigma(alpha)$ is in the fixed field of $G(E/L)$, which is $L$ itself. (Here's where we use the fact that the extension is Galois).



          Now similarly you can prove that $sigma^-1(L) subseteq K$ to finish off the proof.






          share|cite|improve this answer





















            Your Answer




            StackExchange.ifUsing("editor", function ()
            return StackExchange.using("mathjaxEditing", function ()
            StackExchange.MarkdownEditor.creationCallbacks.add(function (editor, postfix)
            StackExchange.mathjaxEditing.prepareWmdForMathJax(editor, postfix, [["$", "$"], ["\\(","\\)"]]);
            );
            );
            , "mathjax-editing");

            StackExchange.ready(function()
            var channelOptions =
            tags: "".split(" "),
            id: "69"
            ;
            initTagRenderer("".split(" "), "".split(" "), channelOptions);

            StackExchange.using("externalEditor", function()
            // Have to fire editor after snippets, if snippets enabled
            if (StackExchange.settings.snippets.snippetsEnabled)
            StackExchange.using("snippets", function()
            createEditor();
            );

            else
            createEditor();

            );

            function createEditor()
            StackExchange.prepareEditor(
            heartbeatType: 'answer',
            convertImagesToLinks: true,
            noModals: false,
            showLowRepImageUploadWarning: true,
            reputationToPostImages: 10,
            bindNavPrevention: true,
            postfix: "",
            noCode: true, onDemand: true,
            discardSelector: ".discard-answer"
            ,immediatelyShowMarkdownHelp:true
            );



            );








             

            draft saved


            draft discarded


















            StackExchange.ready(
            function ()
            StackExchange.openid.initPostLogin('.new-post-login', 'https%3a%2f%2fmath.stackexchange.com%2fquestions%2f2855760%2fprove-that-k-and-l-are-conjugate-iff-ge-k-and-ge-l-are-conjugate-sub%23new-answer', 'question_page');

            );

            Post as a guest






























            1 Answer
            1






            active

            oldest

            votes








            1 Answer
            1






            active

            oldest

            votes









            active

            oldest

            votes






            active

            oldest

            votes








            up vote
            0
            down vote













            I believe that the you need the assumption that $F subseteq E$ is a Galois extension. Indeed consider the non-Galois extension $mathbbQ subseteq E = mathbbQ(sqrt[3]2)$. The Galois group is the trivial group of a single element. Hence $G(E/mathbbQ) = G(E/E)$ and in particular they are conjugates. However it is not true that there exists $sigma in G(E/mathbbQ)$ s.t. $sigma(E) = mathbbQ$, as then $sigma$ must be the identity and we know $E not = mathbbQ$



            If we assume the extension is Galois the claim holds. In particular if $sigma G(E/K)sigma^-1 = G(E/L)$, then we have that $sigma(K) = L$.



            First we'll prove $sigma(K) subseteq L$. Consider $sigma(alpha)$ for some $alpha in K$. Also let $gamma$ be an arbitrary element of $G(E/L)$. Then we have that $exists beta in G(E/K)$ s.t. $gamma = sigma beta sigma^-1$. Then we have:



            $$gamma(sigma(alpha)) = sigma beta sigma^-1(sigma(alpha)) = sigma(alpha)$$



            So we have that $sigma(alpha)$ is in the fixed field of $G(E/L)$, which is $L$ itself. (Here's where we use the fact that the extension is Galois).



            Now similarly you can prove that $sigma^-1(L) subseteq K$ to finish off the proof.






            share|cite|improve this answer

























              up vote
              0
              down vote













              I believe that the you need the assumption that $F subseteq E$ is a Galois extension. Indeed consider the non-Galois extension $mathbbQ subseteq E = mathbbQ(sqrt[3]2)$. The Galois group is the trivial group of a single element. Hence $G(E/mathbbQ) = G(E/E)$ and in particular they are conjugates. However it is not true that there exists $sigma in G(E/mathbbQ)$ s.t. $sigma(E) = mathbbQ$, as then $sigma$ must be the identity and we know $E not = mathbbQ$



              If we assume the extension is Galois the claim holds. In particular if $sigma G(E/K)sigma^-1 = G(E/L)$, then we have that $sigma(K) = L$.



              First we'll prove $sigma(K) subseteq L$. Consider $sigma(alpha)$ for some $alpha in K$. Also let $gamma$ be an arbitrary element of $G(E/L)$. Then we have that $exists beta in G(E/K)$ s.t. $gamma = sigma beta sigma^-1$. Then we have:



              $$gamma(sigma(alpha)) = sigma beta sigma^-1(sigma(alpha)) = sigma(alpha)$$



              So we have that $sigma(alpha)$ is in the fixed field of $G(E/L)$, which is $L$ itself. (Here's where we use the fact that the extension is Galois).



              Now similarly you can prove that $sigma^-1(L) subseteq K$ to finish off the proof.






              share|cite|improve this answer























                up vote
                0
                down vote










                up vote
                0
                down vote









                I believe that the you need the assumption that $F subseteq E$ is a Galois extension. Indeed consider the non-Galois extension $mathbbQ subseteq E = mathbbQ(sqrt[3]2)$. The Galois group is the trivial group of a single element. Hence $G(E/mathbbQ) = G(E/E)$ and in particular they are conjugates. However it is not true that there exists $sigma in G(E/mathbbQ)$ s.t. $sigma(E) = mathbbQ$, as then $sigma$ must be the identity and we know $E not = mathbbQ$



                If we assume the extension is Galois the claim holds. In particular if $sigma G(E/K)sigma^-1 = G(E/L)$, then we have that $sigma(K) = L$.



                First we'll prove $sigma(K) subseteq L$. Consider $sigma(alpha)$ for some $alpha in K$. Also let $gamma$ be an arbitrary element of $G(E/L)$. Then we have that $exists beta in G(E/K)$ s.t. $gamma = sigma beta sigma^-1$. Then we have:



                $$gamma(sigma(alpha)) = sigma beta sigma^-1(sigma(alpha)) = sigma(alpha)$$



                So we have that $sigma(alpha)$ is in the fixed field of $G(E/L)$, which is $L$ itself. (Here's where we use the fact that the extension is Galois).



                Now similarly you can prove that $sigma^-1(L) subseteq K$ to finish off the proof.






                share|cite|improve this answer













                I believe that the you need the assumption that $F subseteq E$ is a Galois extension. Indeed consider the non-Galois extension $mathbbQ subseteq E = mathbbQ(sqrt[3]2)$. The Galois group is the trivial group of a single element. Hence $G(E/mathbbQ) = G(E/E)$ and in particular they are conjugates. However it is not true that there exists $sigma in G(E/mathbbQ)$ s.t. $sigma(E) = mathbbQ$, as then $sigma$ must be the identity and we know $E not = mathbbQ$



                If we assume the extension is Galois the claim holds. In particular if $sigma G(E/K)sigma^-1 = G(E/L)$, then we have that $sigma(K) = L$.



                First we'll prove $sigma(K) subseteq L$. Consider $sigma(alpha)$ for some $alpha in K$. Also let $gamma$ be an arbitrary element of $G(E/L)$. Then we have that $exists beta in G(E/K)$ s.t. $gamma = sigma beta sigma^-1$. Then we have:



                $$gamma(sigma(alpha)) = sigma beta sigma^-1(sigma(alpha)) = sigma(alpha)$$



                So we have that $sigma(alpha)$ is in the fixed field of $G(E/L)$, which is $L$ itself. (Here's where we use the fact that the extension is Galois).



                Now similarly you can prove that $sigma^-1(L) subseteq K$ to finish off the proof.







                share|cite|improve this answer













                share|cite|improve this answer



                share|cite|improve this answer











                answered Jul 19 at 22:50









                Stefan4024

                28.3k53174




                28.3k53174






















                     

                    draft saved


                    draft discarded


























                     


                    draft saved


                    draft discarded














                    StackExchange.ready(
                    function ()
                    StackExchange.openid.initPostLogin('.new-post-login', 'https%3a%2f%2fmath.stackexchange.com%2fquestions%2f2855760%2fprove-that-k-and-l-are-conjugate-iff-ge-k-and-ge-l-are-conjugate-sub%23new-answer', 'question_page');

                    );

                    Post as a guest













































































                    Comments

                    Popular posts from this blog

                    Color the edges and diagonals of a regular polygon

                    Relationship between determinant of matrix and determinant of adjoint?

                    What is the equation of a 3D cone with generalised tilt?